A debt of $9000 is to be amortized with 4 equal semiannual payments. If the interest rate is 7%, compounded semiannually, what is the size of each payment?

Answers

Answer 1

Answer:

Instalment = Loan / PVAF (r%, n)

Where r is int rate per six months & n is no. of half years

= $ 9000 / PVAF (5% , 4)

= $ 9000/ 3.5460

= $ 2538.11

Pls comment, if any further assistance is required.

Step-by-step explanation:


Related Questions

The number of adults who attend a music festival, measured in hundreds of people, is represented by the function a(d)=−0.3d2+3d+10, where d is the number of days since the festival opened.

The number of teenagers who attend the same music festival, measured in hundreds of people, is represented by the function t(d)=−0.2d2+4d+12, where d is the number of days since the festival opened.

What function, f(d) , can be used to determine how many more teenagers than adults attend the festival on any day?


f(d)=−0.1d2+d+22

f(d)=0.1d2+d+2

f(d)=−0.1d2+7d+2

f(d)=0.1d2+7d+2

Answers

Answer:

f(d)=0.1d^2+d+2

Step-by-step explanation:

t(d)=−0.2d2+4d+12

a(d)=−0.3d2+3d+10

how many more teenagers than adults attend the festival on any day?

==>

f(d) = t(d) - a(d)

=0.1d^2+d+2

SOMEONE HELP ME PLEASE
find the real fifth root of -32

Answers

Answer:  -2

This is because (-2)^5 = -32. Applying the fifth root to both sides lets us say [tex]-2 = \sqrt[5]{-32}[/tex]

There are four other roots but they are complex. Effectively, we are solving the equation [tex]x^5 + 32 = 0[/tex]

Which of these statements is correct? The system of linear equations 6 x minus 5 y = 8 and 12 x minus 10 y = 16 has no solution. The system of linear equations 7 x + 2 y = 6 and 14 x + 4 y = 16 has an infinite number of solutions. The system of linear equations 8 x minus 3 y = 10 and 16 x minus 6 y = 22 has no solution. The system of linear equations 9 x + 6 y = 14 and 18 x + 12 y = 26 has an infinite number of solutions

Answers

Answer:

The only true statement is:

"The system of linear equations 8x - 3y = 10 and 16x - 6y = 22 has no solution."

Step-by-step explanation:

First, some definitions.

A system of linear equations has infinite solutions if both equations define the same line, has no solutions if we have two parallel lines, has one solution in all the other cases.

Where two lines are parallel if we can write them as:

a*x + b*y = c

a*x + b*y = d

where c and d are different numbers.

Now we can analyze the given statements:

a)

6x - 5y = 8

12x - 10y = 16

has no solution?

If we divide both sides of the second equation by 2, we get:

(12x - 10y)/2 = 16/2

6x - 5y = 8

We get the first equation, then both equations define the same line, thus the system has infinite solutions, then the statement is false.

b)

7x + 2y = 6

14x + 4y = 16

has infinite solutions?

Let's divide the second equation by 2, then we get:

(14x + 4y)/2 = 16/2

7x + 2y = 8

If we rewrite our system of equations, we get:

7x + 2y = 6

7x + 2y = 8

These are parallel lines, thus, this system has no solutions.

So the statement is false.

c)

8x - 3y = 10

16x - 6y = 22

has no solution?

Again, let's divide the second equation by 2 to get:

(16x - 6y)/2 = 22/2

8x - 3y = 11

If we rewrite our system:

8x - 3y = 10

8x - 3y = 11

These are parallel lines, thus the system has no solutions, so this statement is correct.

d)

9x + 6y = 14

18x + 12y = 26

Has infinite solutions?

Dividing the second equation by 2 we get:

(18x + 12y)/2 = 26/2

9x + 6y = 13

So the equations are different (are parallel lines again) so this system has not infinite solutions.

Then the statement is false.

Answer:

The answer to your question is the third choice.

Step-by-step explanation:

a)          6x - 5y = 8

          12x - 10y = 16

We observe that these lines are the same so they have infinite solutions.

b)

           7x + 2y = 6

          14x + 4y = 16

These lines are parallel because they have the same slope, so they do not cross, there is no solution.

c)

            8x - 3y = 10

           16x - 6y = 22

These lines are parallel because they have the same slope, so they do not cross, there is no solution.

d)

           9x + 6y = 14

          18x + 12y = 26

These lines are parallel because they have the same slope, so they do not cross, they do not have an infinite number of solutions.

Find the surface area of each solid figure

Answers

Answer:

First find the SA of the triangular figure

4 x 3 = 12 cm^2 (the triangles on the sides)

2 x 3 = 6 cm^2 (the back square)

2 x 5 = 10 cm^2 (the slanted square)

*I'm not sure if this question includes the bottom of the triangle but here it is anyways

4 x 2 = 8 cm^2

Including the bottom the SA of the triangular figure is:

12 + 6 + 10 + 8 = 36 cm^2

Find the SA of the rectangular shape

4 x 2 = 8 cm^2 (the bottom square)

2 x 6 = 12 x 2 = 24 cm^2 (the sides)

4 x 6 = 24 x 2 = 48 cm^2 (the front and back)

Add them up

8 + 24 + 48 = 80 cm^2

If you wanted to find the SA of the whole figure it would be:

12 + 6 + 10 + 8 + 24 + 48 = 108 cm^2

Hope this helps!

Patel squeezed oranges so that his family could have fresh-squeezed juice for breakfast. He squeezed StartFraction 4 over 17 EndFraction cups from the first orange, StartFraction 3 over 10 EndFraction cups from the second orange, StartFraction 9 over 20 EndFraction cups from the third orange, StartFraction 3 over 11 EndFraction cups from the fourth orange, and StartFraction 7 over 15 EndFraction cups from the fifth orange. Patel estimates that he needs 3 cups of orange juice for his family. About how much more orange juice does he need to reach his estimate?

Answers

Answer:

A. 1/2 cups

Step-by-step explanation:

13/15 is close to 1

1/5 is a small amount

9/20 is just over 1/2

5/11 is just under 1/2

7/15 is just under 1/2

Estimate: 1 + 1/2 + 1/2 + 1/2 + a little = 2 1/2

He needs 3 cups, so he needs another 1/2 cup.

Answer: A. 1/2 cups

Answer:

it a 1/2

Step-by-step explanation:

WHAT is the answer? Pls Time limit

Answers

Answer:

11 is the answer.....

Step-by-step explanation:

6x-15+39+90 = 180

I need the answer. Please help me

Answers

C = 25 deg

a ≈ 10.72 ft

b ≈ 11.83 ft

Answer:

C = 25

a = 10.72

b = 11.83

Step-by-step explanation:

C:

Solve; 180 - (90 + 65)

a:

Tan(65) = a/5

Tan(65) * 5 = a

10.72 = a

b:

Cos(65) = 5/b

Cos(65) * b = 5

b = 5 / Cos(65)

b = 11.83

Hope this helps!

Question 4 of 10
Which of the following formulas would find the lateral area of a right cylinder
with height equal to hand r as the radius?
A. LA = 2012 + 2q rh
B. LA = 20
C. LA = 2 r rh
D. LA = 27.12
SUBMIT

Answers

Answer:

LA = 2πrh

Step-by-step explanation:

The lateral surface area is the area of all the sides of a 3-dimensional object excluding its base and top.

The lateral area of a cylinder is the area of its surfaces excluding the area of its base and top

LA = 2πrh

Use the slope formula to find the slope of the line through the points (2,10) and (10,−8).

Answers

The slope formula is the changes of two y-values over/to the changes of two x-values.

[tex] \large \boxed{m = \frac{y_2 - y_1}{x_2 - x_1} }[/tex]

Substitute two given points in the formula to find the slope. The m-term represents the slope from y = mx+b.

[tex]\large{m = \frac{10 - ( - 8)}{2 - 10} } \\ \large{m = \frac{10 + 8}{ - 8} } \\ \large{ m = \frac{18} { - 8} \longrightarrow \frac{9}{ - 4} } \\ \large \boxed{m = - \frac{9}{4} }[/tex]

Answer

The slope is -9/4.

Hope this helps and let me know if you have any doubts!

Answer:

m=-9/4

Step-by-step explanation:

Hi there!

The formula for the slope (m) calculated from two points is given as (y2-y1)/(x2-x1), where (x1,y1) and (x2,y2) are points

we are given the two points (2,10) and (10,-8)

to avoid any confusion, let's label the values of the points

x1=2

y1=10

x2=10

y2=-8

now substitute into the formula:

m=(-8-10)/(10-2)

subtract

m=(-18)/(8)

simplify (reduce to lowest terms)

m=-9/4

Hope this helps!

PLEASE WILL MARK IF YOU HELP!!

Answers

Answer:

22°

63°

m<H=22°

m<G=63°

Problem 1

Answer:   79

--------------------------

Work Shown:

For any triangle, the three angles always add to 180

For any isosceles triangle, the base angles are congruent. The base angles are opposite the congruent sides. We see that angle O = angle H.

O+H+T = 180

H+H+T = 180

2H+T = 180

2H+22 = 180

2H = 180 - 22

2H = 158

H = 158/2

H = 79

=======================================================

Problem 2

Answer:   54

--------------------------

Work Shown:

We'll use the same ideas as problem 1.

In this case, angle O = angle D = 63 since they are the base angles opposite the congruent sides.

D+G+O = 180

63+G+63 = 180

G+126 = 180

G = 180-126

G = 54

What is the mean of the data?

Answers

Answer:

The mean (average) of a data set is found by adding all numbers in the data set and then dividing by the number of values in the set. The median is the middle value when a data set is ordered from least to greatest. The mode is the number that occurs most often in a data set.

Use implicit differentiation to find an equation of the tangent line to the curve at the given point. y2(y2 − 4) = x2(x2 − 5) (0, −2) (devil's curve)

Answers

Answer:

Step-by-step explanation:

Given that:

[tex]y^2 (y^2-4) = x^2(x^2 -5)[/tex]

at point (0, -2)

[tex]\implies y^4 -4y^2 = x^4 -5x^2[/tex]

Taking the differential from the equation above with respect to x;

[tex]4y^3 \dfrac{dy}{dx}-8y \dfrac{dy}{dx}= 4x^3 -10x[/tex]

Collect like terms

[tex](4y^3 -8y)\dfrac{dy}{dx}= 4x^3 -10x[/tex]

[tex]\dfrac{dy}{dx}= \dfrac{4x^3 -10x}{4y^3-8y}[/tex]

Hence, the slope of the tangent line m can be said to be:

[tex]\dfrac{dy}{dx}= \dfrac{4x^3 -10x}{4y^3-8y}[/tex]

At point (0,-2)

[tex]\dfrac{dy}{dx}= \dfrac{4(0)^3 -10(0)}{4(-2)^3-8-(2)}[/tex]

[tex]\dfrac{dy}{dx}= \dfrac{0 -0}{4(-8)+16}[/tex]

[tex]\dfrac{dy}{dx}= 0[/tex]

m = 0

So, we now have the equation of the tangent line with slope m = 0 moving through the point (x, y) = (0, -2) to be:

(y - y₁ = m(x - x₁))

y + 2 = 0(x - 0)

y + 2 = 0

y = -2

Lori downloaded all the pictures she took at Rita’s wedding into a single computer folder. She took 86 of the 134 pictures with her camera and the remainder of them with her cell phone. Of the pictures Lori took with her cell phone, one out of every five was blurry.

Answers

Answer:

87

Step-by-step explanation:

what’s “24 increased by a number y is 41” as an equation?

Answers

Answer:

24 + y = 41

y = 17

Step-by-step explanation:

24 + y = 41

y + 24 = 41

y + 24 - 24 = 41 - 24

y = 17

You deposit $10,000 in an account earning 4% interest compounded monthly. a. How much will you have in the account in 25 years? b. How much interest will you earn?

Answers

Answer:

In 25 years I will have $ 27,137.65. Therefore, I will earn $17,137.65 interest.

Step-by-step explanation:

Given that I deposit $ 10,000 in an account earning 4% interest compounded monthly, to determine how much will you have in the account in 25 years and how much interest will I earn, the following calculation must be performed:

10,000 x (1 + 0.04 / 12) ^ 25x12 = X

10,000 x (1 + 0.00333) ^ 300 = X

10,000 x 2.7137 = X

27,137.65 = X

Therefore, in 25 years I will have $ 27,137.65. Therefore, I will earn $17,137.65 interest.

PLEASE HELP ME I HAVE TO PASS THIS TEST

30 POINTS

Answers

Answer:

Hi, there the answer is

These are the equations with exactly one solution

-5x + 12 = –12x – 12

-5x + 12 = 5x + 12

-5x + 12 = 5x – 5

Hope This Helps :)

Step-by-step explanation:

First degree equations

A first degree equation has the form

ax + b = 0

There are some special cases where the equation can have one, infinitely many or no solution

If , the equation has exactly one solution

If a=0 and b=0 the equation has infinitely many solutions, because it doesn't matter the value of x, it will always be true that 0=0

If a=0 and  the equation has no solution, because it will be equivalent to b=0 and we are saying it's not true. No matter what x is, it's a false statement.

We have been given some equations, we only need to put them in standard form

-5x + 12 = –12x – 12

Rearranging

7x + 24 = 0

It has exactly one solution because a is not zero

.......................

-5x + 12 = 5x + 12

Rearranging

-10x + 0 = 0

It has exactly one solution because a is not zero

.......................

-5x + 12 = 5x – 5

Rearranging

-10x + 17 = 0

It has exactly one solution because a is not zero

.......................

-5x + 12 = -5x – 12

Rearranging

0x + 24 = 0

It has no solution, no matter what the value of x is, it's impossible that 24=0

Answer: These are the equations with exactly one solution

-5x + 12 = –12x – 12

-5x + 12 = 5x + 12

-5x + 12 = 5x – 5

Hi the answer is -5x+12=-12x-12

What is the solution to this inequality?

14 + x ≤ 26

Answers

Answer:

[tex]14 + x \leqslant 26 \\ x \leqslant 26 - 14 \\ { \tt{x \leqslant 12}}[/tex]

PLEASE HELP!!!

WILL MARK BRAINLIEST!!!

If the diameter of the circle shown below is 6ft and 0 is a right angle, what is the length of segment AB to the nearest foot?

Multiple choice!

Thank you!

Answers

Answer:

how old are you gghhjjzetstu9u

Answer:

4 ft

Step-by-step explanation:

let's find radius first

radius=diameter/2

=6/2

=3 ft

radii=3 ft

Now by using pythagoras theorem

a^2 + b^2 = c^2

3^2 + 3^2 =AB^2

9+9=AB^2

18=AB^2

[tex]\sqrt{18}[/tex] AB

4.24 =AB

4 ft =AB (after converting to nearest foot)

Jai bought a helmet and a pair of skates.
The helmet cost £45.
He sold both items for £224.
Jai made a 120% profit on the cost of the helmet and a 40% profit on the total cost.
What was the percentage profit on the skates?
Give your answer to 1 decimal place.

Answers

Answer:

Profit % on skates = 8.7 %

Step-by-step explanation:

Step 1 : Find cost price of skates

Cost price of helmet = £45

Let cost price of skate be = x

Selling price = £224

Cost price = (x + 45)

Total profit % = 40%

[tex]Profit \% = \frac{Selling \ price - cost \ price }{Cost \ price} \times 100[/tex]

[tex]\frac{40}{100} = \frac{224 - (x + 45)}{(x + 45)}\\\\40(x+ 45) = 100(224 - (x +45))\\\\40(x + 45) = 22400 - 100(x + 45)\\\\40(x +45) + 100(x+ 45) = 22400\\\\140(x + 25) = 22400\\\\x + 45 = \frac{22400}{140}\\\\x = 160 - 45 = \£ \ 115[/tex]

Total cost price = 45 + 115 = £160

Step 2 : Selling price of Helmet

Cost price of Helmet = £45

Let selling price of helmet be = y

Profit % of helmet = 120 %

[tex]Profit \% = \frac{selling \ price - cost \ price}{cost \ price}[/tex]

[tex]\frac{120}{100} = \frac{y -45}{45}\\\\\frac{120 \times 45}{100} = y -45\\\\54 = y - 45\\\\99 = y[/tex]

Step 3 : Selling price of skates

Total selling = selling price of helmet + selling price of skates

224 = 99 + selling price of skates

224 - 99 = selling price of skates

125 = selling price of skates

Step 4 : Profit percentage on skates

Cost price of skate = £ 115

Selling price of skate = £ 125

[tex]Profit \% \ on \ skates = \frac{selling\ price- cost \ price }{cost \ price} \times 100[/tex]

                           [tex]= \frac{125-115}{115} \times 100\\\\=\frac{10}{115} \times 100\\\\= 8.7 \%[/tex]

What is the zero of the function represented by this graph?

Answers

answer: -5
explanation: the x-intercept is (-5,0), the x value of the x intercept is the zero.

Longhorn Pizza has the following number of topping options available: four vegetables, two meats, and two cheeses. A pizza is ordered with exactly four toppings. What is the probability that the pizza is ordered with exactly two vegetables, one meat, and one cheese

Answers

Answer:

The probability is [tex]\frac{24}{70}[/tex].

Step-by-step explanation:

topping options available: four vegetables, two meats, and two cheeses

Number of topping on one pizza = 4

Getting two vegetables = (4 C 2)

Getting one meat = (2 C 1)

Getting one cheese = (2 C 1)

Choosing 4 toppings out of 8 = (8 C 4)

probability that the pizza is ordered with exactly two vegetables, one meat, and one cheese

[tex]\frac{(4C2)\times (2C1)\times (2C1)}{(8C4)}\\\\\frac{6\times 2\times 2}{70}\\\\\frac{24}{70}[/tex]

The endpoints of DEF are D(1, 4) and F(16, 14).
Determine and state the coordinates of point E, if
DE: EF = 2:3.

Answers

Answer:

The coordinates of point E are (7,8).

Step-by-step explanation:

Point E:

Is given by (x,y).

DE: EF = 2:3.

This means that, for both coordinates x and y:

[tex]E - D = \frac{2}{2+3}(F-D)[/tex]

[tex]E - D = \frac{2}{5}(F-D)[/tex]

x-coordinate:

x-coordinate of D: 1

x-coordinate of F: 16

[tex]E - D = \frac{2}{5}(F-D)[/tex]

[tex]x - 1 = \frac{2}{5}(16-1)[/tex]

[tex]x - 1 = 2*3[/tex]

[tex]x = 7[/tex]

y-coordiante:

y-coordinate of D: 4

y-coordinate of F: 14

[tex]E - D = \frac{2}{5}(F-D)[/tex]

[tex]y - 4 = \frac{2}{5}(14-4)[/tex]

[tex]y - 4 = 2*2[/tex]

[tex]x = 8[/tex]

The coordinates of point E are (7,8).

Please help show the steps
Please put 15 years old

Answers

Answer:

P = $98.77

Step-by-step explanation:

FV = p (1+i)^n -1

            i

pv = 700,000

i = .075/12 = .00625

n = (66 - 15)* 12 = 612

700,000 = P (( 1 + .00625)^ 612 -1 /.00625

4375 = P (1.00625)^612 -1)

P = $98.77

Answer:

page 1:

51 years

$98.78

639546.64 (i think)

Page 2:

213 months

17.8 years

321 months

26.8 years

1128.9 months

88.8 years

I would probably choose the second plan because it's rather unlikely that i live past 90

Step-by-step explanation:

page 1

Let's assume the payments are at the end of the month

66-15= 51 years

effective rate: .075/12=.00625

[tex]700000=x\frac{(1+.00625)^{51*12}-1}{.00625}\\x=98.77973387[/tex]

which i guess we can round to 98.78

700000-98.78*(51*12)= 639546.64

This number is really really high and so maybe you want to double check it

page 2

effective rate: .051/12=.00425

[tex]700000=5000\frac{1-(1+.00425)^{-n}}{.00425}\\.405=(1+.00425)^{-n}\\log_{1.00425}.405=-n\\n=213[/tex]

213 months

213/12= 17.8 years

[tex]700000=4000\frac{1-(1+.00425)^{-n}}{.00425}\\.25625=(1.00425)^{-n}\\log_{1.00425}.25625\\n=321[/tex]

321 months

321/12=26.8 years

[tex]700000=3000\frac{1-(1+.00425)^{-n}}{.00425}\\.008333333=(1.0045)^{-n}\\log_{1.0045}.00833333=-n\\n=1128.9[/tex]

1128.9 months

1128.9/12= 94.1 years

1066 months

1066/12= 88.8 years

A golfer hits a golf ball.
The function
d(t) = –2t2 + 7t + 4
most closely represents the height(h) of the golf ball in feet after t seconds. How
long is the golf ball in the air?

Answers

Answer:

The golf ball was in the air for 4 seconds.

Step-by-step explanation:

Solving a quadratic equation:

Given a second order polynomial expressed by the following equation:

[tex]ax^{2} + bx + c, a\neq0[/tex].

This polynomial has roots [tex]x_{1}, x_{2}[/tex] such that [tex]ax^{2} + bx + c = a(x - x_{1})*(x - x_{2})[/tex], given by the following formulas:

[tex]x_{1} = \frac{-b + \sqrt{\Delta}}{2*a}[/tex]

[tex]x_{2} = \frac{-b - \sqrt{\Delta}}{2*a}[/tex]

[tex]\Delta = b^{2} - 4ac[/tex]

In this question:

We have to find the amount of time it takes for the ball to hit the ground. We have that:

[tex]d(t) = -2t^2 + 7t + 4[/tex]

Which is a quadratic equation with [tex]a = -2, b = 7, c = 4[/tex].

How long is the golf ball in the air?

We have to find t for which [tex]d(t) = 0[/tex]

So

[tex]-2t^2 + 7t + 4 = 0[/tex]

[tex]\Delta = b^{2} - 4ac = (7)^2 - 4(-2)(4) = 81[/tex]

[tex]t_{1} = \frac{-7 + \sqrt{81}}{2*(-2)} = -0.5[/tex]

[tex]t_{2} = \frac{-7 - \sqrt{81}}{2*(-2)} = 4[/tex]

Time is a positive measure, so t = 4.

The golf ball was in the air for 4 seconds.

Find all real zeros of the function y = -7x + 8

Answers

9514 1404 393

Answer:

  x = 8/7

Step-by-step explanation:

The only real zero of this linear function is the value of x that makes y=0:

  0 = -7x +8

  7x = 8 . . . . . . add 7x

  x = 8/7 . . . . . .divide by 7

m.ng giúp mình về phần vector trong ma trận nha

Answers

Answer:

maybe if u translate it in English

Step-by-step explanation:

it wouldv been helpful if u mind?

Find the area of this circle. Use 3 for T.
Α = πη2
5 in
[?] in?

Answers

Hope this help!!!

Have a nice day!!!

A
2x+5
x² + 5x + 6
x² + 5x+6
B
2x+5​

Answers

Answer:

what is the question?

Step-by-step explanation:

answer the question

Find the equation of a line with a slope of −1/2 that passes through the point −4, 10

Answers

Answer:

y - 10 = -1/2(x + 4)

General Formulas and Concepts:

Algebra I

Coordinates (x, y)

Point-Slope Form: y - y₁ = m(x - x₁)

x₁ - x coordinate y₁ - y coordinate m - slope

Step-by-step explanation:

Step 1: Define

Identify variables

m = -1/2

Point (-4, 10) → x₁ = -4, y₁ = 10

Step 2: Find

Substitute in variables [Point-Slope Form]:                                                    y - 10 = -1/2(x - -4)Simplify:                                                                                                             y - 10 = -1/2(x + 4)

Answer: [tex]y=-\frac{1}{2} x+8[/tex]

Step-by-step explanation:

An equation of a line can be in slope intercept form which is y=mx+b

m is the slope, b is the y intercept, x it the x coordinate, and y is the y coordinate. Since we know the slope is -1/2 and we know a x coordinate is -4 and a y coordinate is 10 we can sub them in and solve for the value of b.

[tex]y=mx+b\\10=(-\frac{1}{2})(-4)+b\\10=2+b\\10-2=2+b-2\\8=b[/tex]

The value of b is 8. We can now sub it in for our equation of the line. This time with x and y as variables.

y=-1/2x+8

3. Find the value of X (In the picture) (giving points to best answer/brainlest)​

Answers

Answer:

101 =x

Step-by-step explanation:

The measure of the exterior angle is equal to the sum of the opposite interior angles

143 = 42+x

Subtract 42 from each side

143-42 = 42+x-42

101 =x

Answer:

x = 101 degrees

Step-by-step explanation:

The sum of the external angle and its adjacent is 180 degrees

143 + y = 180

y = 37 degrees

The sum of the inner angles of a triangle is 180 degrees

37 + 42 + x = 180

79 + x = 180

x = 101 degrees

Other Questions
624,612 rounded to the nearest hundred is Jamie answered 68 out of 80 questions. What percentage represents the point number of questions she answered? Given that abc and def are similar find fd Your car breaks down in the middle of nowhere. A tow truck weighing 4000 lbs. comes along and agrees to tow your car, which weighs 2000 lbs., to the nearest town. The driver of the truck attaches his cable to your car at an angle of 20 degrees to horizontal. He tells you that his cable has a strength of 500 lbs. He plans to take 10 secs to tow your car at a constant acceleration from rest in a straight line along a flat road until he reaches the maximum speed of 45 m.p.h. Can the driver carry out the plan Need help with math quarterly please Which development best illustrates why world war I is classified as a total war? Suppose that today you buy a bond with an annual coupon rate of 10 percent for $1,120. The bond has 17 years to maturity. What rate of return do you expect to earn on your investment What is the probability that a student selected at random is a boy? P( AnswerB )= AnswerWhat is the probability that a boy selected at random is a Junior?P( AnswerJ | B )= AnswerWhat is the probability of randomly choosing a student who is a Junior and a boy?P(J Answer B)= AnswerWhat is the probability of randomly choosing a student who is a girl given that she is not junior?P( AnswerG | J ' )= AnswerWhat is the probability of randomly selecting a girl? P(G)= Answer pls help. will mark brainliest. if U= {1,2,3,4,5,6}, A ={1,2,3,}B={2,3,5},and C ={4,5,6}find (AUB)'n c The relative frequency table below summarizes a survey about support for candidate X in an upcomingnational election. In this survey, 210 randomly selected Virginians opposed candidate X.How many people in this survey were Californians who opposed candidate X?Does the table show evidence of an association between being from Virginia and opposing candidate X?Choose all answers that apply: Today is Leilas birthday. She is 5 years old. How old is leila in months Please guys help me fast!!!!! In a sample of oxygen gas at room temperature, the average kinetic energy of all the balls stays constant. Which postulate of kinetic molecular theory best explainshow this is possible?Attractive forces between gas particles are negligible because the particles of an ideal gas are moving so quicklyO Collisions between gas particles are elastic, there is no net gain or loss of kinetic energyGases consist of a large number of small particles, with a lot of space between the particlesO Gas particles are in constant, random motion, and higher kinetic energy means faster movement A staff member at UF's Wellness Center is interested in seeing if a new stress reduction program will lower employees systolic blood pressure levels. Twenty people are selected and have their blood pressure measured. Each person then participates in the stress reduction program. One month after the stress reduction program, the systolic blood pressure levels of the employees were measured again. Did the program reduce the average systolic blood pressure level? Note: Differences were taken by taking "before systolic blood pressure" minus "After blood pressure"The 95% confidence interval was (5.6, 10.2).A. On average, the blood pressure levels before the stress reduction program were significantly higher than the systolic blood pressure levels after the stress reduction program, at the 95% confidence levelB. We are 95% confident that there is no difference in average systolic blood pressure levels before and after the stress reduction program.C. On average, the systolic blood pressure levels after the stress reduction program were significantly higher than the systolic blood pressure levels before the stress reduction program, at the 95% confidence level. Trevor had 3/4 pound of peanut she wants to split them evenly among 4 people. How many pounds of peanuts would each person get? help me find the volume (picture) WHich of these foods are often marketed as healthy but may not be as good for you as advertsers claim This question is from the similarity chapter. It would be really kind of you if you would answer this question. When subtracting 1.578 from 127.3, the result on the calculator is 125.722. Round the answer to the appropriate precision based on the precision of the numbers in the subtraction column